wu :: forums
« wu :: forums - Consecutive integer products »

Welcome, Guest. Please Login or Register.
May 13th, 2024, 4:04pm

RIDDLES SITE WRITE MATH! Home Home Help Help Search Search Members Members Login Login Register Register
   wu :: forums
   riddles
   medium
(Moderators: william wu, towr, SMQ, Icarus, Grimbal, ThudnBlunder, Eigenray)
   Consecutive integer products
« Previous topic | Next topic »
Pages: 1  Reply Reply Notify of replies Notify of replies Send Topic Send Topic Print Print
   Author  Topic: Consecutive integer products  (Read 356 times)
NickH
Senior Riddler
****





   
WWW

Gender: male
Posts: 341
Consecutive integer products  
« on: Jun 7th, 2003, 3:16am »
Quote Quote Modify Modify

Show that each of the following equations has no solution in integers x > 0, y > 0, n > 1.

    1. x(x + 1) = yn
    2. x(x + 1)(x + 2) = yn
    3. x(x + 1)(x + 2)(x + 3) = yn

Does the pattern continue?  (I don't know the answer to this one.)
IP Logged

Nick's Mathematical Puzzles
Sir Col
Uberpuzzler
*****




impudens simia et macrologus profundus fabulae

   
WWW

Gender: male
Posts: 1825
Re: Consecutive integer products  
« Reply #1 on: Jun 7th, 2003, 6:38am »
Quote Quote Modify Modify

The generalisation of this result is a very difficult problem. It took Erdos nine years to crack! You may like to check out this link: http://www.nrich.maths.org.uk/askedNRICH/edited/3945.html
 
I've made no significant advances with extending the particular cases to the general case, but I'll share it anyway...
 
::
Obviously with the first case we know that HCF(x,x+1)=1, so clearly this cannot be a perfect power. That is, the highest factor of the LHS, x(x+1), will be 1, and the highest factor of the RHS, yn, will be y. Therefore no solutions exist for y>1 and clearly there are no solutions for y=1.
 
In a similar way, HCF(x,x+1,x+2)=2, so for y>2, x(x+1)(x+2)=yn will have no integral solutions. Checking the cases y=1,2, we confirm that there are no positive integer solutions at all. The same idea can be exteneded to x(x+1)(x+2)(x+3)=yn. The HCF of the LHS will be 3, so there will be no integral solutions for y>3. Checking, confirms that no solutions exist for y=1,2,3.
 
You can see (and which is why NickH probably stopped here), extending this to the general case by this method will fail. We can agree that for x(x+1)(x+2)...(x+k), the highest common factor will be k and so no solutions exist for y>k, however, it is not possible to confirm this with the cases y=1,2,3,...,k-1.
 
I feel that it'd be nice to use a different approach, so here are some of my (failed) ramblings with the first case.
 
As x and (x+1) are consecutive integers, their product, yn, must be even and as n>1, it must contain at least a factor of 4.
 
1+2+3+...+x=x(x+1)/2, so yn=2(1+2+...+x). As LHS has a factor of 4, the sum, 1+2+...+x, must be even. Therefore, if a solution exists, x must be odd.
 
We know that yn==0 mod 4. However, x(x+1)=x2+x and if x is odd, x2==1 mod 4, which means that x==3 mod 4; that is, x=4k–1.
 
That's it for the moment. I suspect in the end that it will be necessary to consider unique prime factorisation anyway.
::
« Last Edit: Jun 7th, 2003, 8:43am by Sir Col » IP Logged

mathschallenge.net / projecteuler.net
Pages: 1  Reply Reply Notify of replies Notify of replies Send Topic Send Topic Print Print

« Previous topic | Next topic »

Powered by YaBB 1 Gold - SP 1.4!
Forum software copyright © 2000-2004 Yet another Bulletin Board